Bạn chưa đăng nhập. Vui lòng đăng nhập để hỏi bài

Những câu hỏi liên quan
Văn Quyết
Xem chi tiết
hiền nguyễn
Xem chi tiết
Nguyễn Lê Phước Thịnh
19 tháng 4 2023 lúc 10:59

\(\dfrac{\sqrt{1+x^3+y^3}}{xy}>=\sqrt{\dfrac{3}{xy}}\)

\(\dfrac{\sqrt{1+y^3+z^3}}{yz}>=\sqrt{\dfrac{3}{yz}}\)

\(\dfrac{\sqrt{1+z^3+x^3}}{xz}>=\sqrt{\dfrac{3}{xz}}\)

=>\(VT>=\sqrt{3}\left(\dfrac{1}{\sqrt{xy}}+\dfrac{1}{\sqrt{yz}}+\dfrac{1}{\sqrt{xz}}\right)=3\sqrt{3}\)

Đỗ Thị Ánh Nguyệt
Xem chi tiết
bac luu
Xem chi tiết
Adu vip
Xem chi tiết
Nguyễn Huy Tú
14 tháng 7 2021 lúc 9:19

undefined

An Thy
14 tháng 7 2021 lúc 9:21

\(\dfrac{1}{x}+\dfrac{1}{y}+\dfrac{1}{z}\ge\dfrac{1}{\sqrt{xy}}+\dfrac{1}{\sqrt{yz}}+\dfrac{1}{\sqrt{zx}}\)

\(\Rightarrow\dfrac{2}{x}+\dfrac{2}{y}+\dfrac{2}{z}\ge\dfrac{2}{\sqrt{xy}}+\dfrac{2}{\sqrt{yz}}+\dfrac{2}{\sqrt{zx}}\)

\(\Rightarrow\dfrac{2}{x}+\dfrac{2}{y}+\dfrac{2}{z}-\dfrac{2}{\sqrt{xy}}+\dfrac{2}{\sqrt{yz}}+\dfrac{2}{\sqrt{zx}}\ge0\)

\(\Rightarrow\dfrac{1}{x}-\dfrac{2}{\sqrt{xy}}+\dfrac{1}{y}+\dfrac{1}{y}-\dfrac{2}{\sqrt{yz}}+\dfrac{1}{z}+\dfrac{1}{z}-\dfrac{2}{\sqrt{zx}}+\dfrac{1}{x}\ge0\)

\(\Rightarrow\left(\dfrac{1}{\sqrt{x}}-\dfrac{1}{\sqrt{y}}\right)^2+\left(\dfrac{1}{\sqrt{y}}-\dfrac{1}{\sqrt{z}}\right)^2+\left(\dfrac{1}{\sqrt{z}}-\dfrac{1}{\sqrt{x}}\right)^2\ge0\) (luôn đúng)

Dấu = xảy ra khi \(x=y=z\)

Dưa Hấu
14 tháng 7 2021 lúc 9:21

undefined

Thiều Khánh Vi
Xem chi tiết
Akai Haruma
25 tháng 9 2018 lúc 23:33

Lời giải:

Đặt \((\sqrt{x}, \sqrt{y}, \sqrt{z})=(a,b,c)\Rightarrow abc=1\)

Bài toán trở thành chứng minh:

\(\frac{1}{(ab+a+1)^2}+\frac{1}{(bc+b+1)^2}+\frac{1}{(ca+c+1)^2}\geq \frac{1}{3}\)

------------

Áp dụng 1 kết quả quen thuộc của BĐT AM-GM: \(x^2+y^2+z^2\geq \frac{(x+y+z)^2}{3}\) ta có:

\(\frac{1}{(ab+a+1)^2}+\frac{1}{(bc+b+1)^2}+\frac{1}{(ca+c+1)^2}\geq \frac{1}{3}\left(\frac{1}{ab+a+1}+\frac{1}{bc+b+1}+\frac{1}{ca+c+1}\right)^2\)

Mà:

\(\frac{1}{ab+a+1}+\frac{1}{bc+b+1}+\frac{1}{ca+c+1}=\frac{c}{abc+ac+c}+\frac{ac}{bc.ac+b.ac+ac}+\frac{1}{ac+c+1}\)

\(=\frac{c}{1+ac+c}+\frac{ac}{c+1+ac}+\frac{1}{ac+c+1}=\frac{ac+c+1}{ac+c+1}=1\) (thay $abc=1$)

Do đó:

\(\frac{1}{(ab+a+1)^2}+\frac{1}{(bc+b+1)^2}+\frac{1}{(ca+c+1)^2}\geq \frac{1}{3}.1^2=\frac{1}{3}\) (đpcm)

Dâu bằng xảy ra khi $a=b=c=1$ hay $x=y=z=1$

Big City Boy
Xem chi tiết
Lấp La Lấp Lánh
31 tháng 10 2021 lúc 12:05

ĐKXĐ: \(\left\{{}\begin{matrix}x\ge0\\y\ge0\\z\ge0\end{matrix}\right.\)

\(A=\dfrac{\sqrt{x}}{\sqrt{xy}+\sqrt{x}+2}+\dfrac{\sqrt{y}}{\sqrt{yz}+\sqrt{y}+1}+\dfrac{2\sqrt{z}}{\sqrt{xz}+2\sqrt{z}+2}\)

\(=\dfrac{\sqrt{x}}{\sqrt{xy}+\sqrt{x}+\sqrt{xyz}}+\dfrac{\sqrt{y}}{\sqrt{yz}+\sqrt{y}+1}+\dfrac{\sqrt{xyz}.\sqrt{z}}{\sqrt{xz}+\sqrt{xyz}.\sqrt{z}+\sqrt{xyz}}\)

\(=\dfrac{1}{\sqrt{yz}+\sqrt{y}+1}+\dfrac{\sqrt{y}}{\sqrt{yz}+\sqrt{y}+1}+\dfrac{\sqrt{yz}}{\sqrt{yz}+\sqrt{y}+1}\)

\(=\dfrac{\sqrt{yz}+\sqrt{y}+1}{\sqrt{yz}+\sqrt{y}+1}=1\)

\(\Rightarrow\sqrt{A}=\sqrt{1}=1\)

Kimian Hajan Ruventaren
Xem chi tiết
Nguyễn Việt Lâm
16 tháng 1 2021 lúc 20:47

\(P\ge\dfrac{\sqrt{3\sqrt[3]{x^3y^3}}}{xy}+\dfrac{\sqrt{3\sqrt[3]{y^3z^3}}}{yz}+\dfrac{\sqrt{3\sqrt[3]{z^3x^3}}}{zx}\)

\(P\ge\sqrt{3}\left(\dfrac{1}{\sqrt{xy}}+\dfrac{1}{\sqrt{yz}}+\dfrac{1}{\sqrt{zx}}\right)\ge\sqrt{3}.3\sqrt[3]{\dfrac{1}{\sqrt{xy.yz.zx}}}=3\sqrt{3}\)

Dấu "=" xảy ra khi \(x=y=z=1\)

tthnew
16 tháng 1 2021 lúc 20:50

Ta có bất đẳng thức sau \(x^3+y^3\ge xy\left(x+y\right)\Leftrightarrow\left(x+y\right)\left(x-y\right)^2\ge0.\)

Do đó:

\(P=\sum\dfrac{\sqrt{1+x^3+y^3}}{xy}\ge\sum\dfrac{\sqrt{xyz+xy\left(x+y\right)}}{xy}\)

\(=\sqrt{x+y+z}\left(\dfrac{1}{\sqrt{xy}}+\dfrac{1}{\sqrt{yz}}+\dfrac{1}{\sqrt{zx}}\right)\ge\sqrt{3\sqrt[3]{xyz}}\cdot3\sqrt[3]{\dfrac{1}{\sqrt{xy}}\cdot\dfrac{1}{\sqrt{yz}}\cdot\dfrac{1}{\sqrt{zx}}}=3\sqrt{3}\)

Đẳng thức xảy ra khi $x=y=z=1.$

Hồng Phúc
16 tháng 1 2021 lúc 20:51

Áp dụng BĐT AM-GM:

\(P=\dfrac{\sqrt{1+x^3+y^3}}{xy}+\dfrac{\sqrt{1+y^3+z^3}}{yz}+\dfrac{\sqrt{1+z^3+x^3}}{zx}\)

\(\ge\dfrac{\sqrt{3xy}}{xy}+\dfrac{\sqrt{3yz}}{yz}+\dfrac{\sqrt{3zx}}{zx}\)

\(=\sqrt{3}\left(\dfrac{1}{\sqrt{xy}}+\dfrac{1}{\sqrt{yz}}+\dfrac{1}{\sqrt{zx}}\right)\)

\(\ge\sqrt{3}.3\sqrt[3]{\dfrac{1}{xyz}}=3\sqrt{3}\)

\(minP=3\sqrt{3}\Leftrightarrow x=y=z\)

Nguyễn Hoàng Minh
Xem chi tiết
Nguyễn Việt Lâm
26 tháng 3 2022 lúc 15:39

\(\sqrt{x+yz}=\sqrt{x\left(x+y+z\right)+yz}=\sqrt{\left(x+y\right)\left(z+x\right)}\ge\sqrt{\left(\sqrt{xz}+\sqrt{xy}\right)^2}=\sqrt{xy}+\sqrt{xz}\)

\(\Rightarrow\dfrac{x}{x+\sqrt{x+yz}}\le\dfrac{x}{x+\sqrt{xy}+\sqrt{xz}}=\dfrac{\sqrt{x}}{\sqrt{x}+\sqrt{y}+\sqrt{z}}\)

Tương tự:

\(\dfrac{y}{y+\sqrt{y+xz}}\le\dfrac{\sqrt{y}}{\sqrt{x}+\sqrt{y}+\sqrt{z}}\)

\(\dfrac{z}{z+\sqrt{z+xy}}\le\dfrac{\sqrt{z}}{\sqrt{x}+\sqrt{y}+\sqrt{z}}\)

Cộng vế:

\(VT\le\dfrac{\sqrt{x}+\sqrt{y}+\sqrt{z}}{\sqrt{x}+\sqrt{y}+\sqrt{z}}=1\)

Dấu "=" xảy ra khi \(x=y=z=\dfrac{1}{3}\)